You are on page 1of 25

Solutions to the November 2012 Course MLC Examination by Krzysztof Ostaszewski, http://www.krzysio.net, krzysio@krzysio.

net Copyright 2012 by Krzysztof Ostaszewski All rights reserved. No reproduction in any form is permitted without explicit permission of the copyright owner. Dr. Ostaszewskis manual for Course MLC is available from Actex Publications (http://www.actexmadriver.com) and the Actuarial Bookstore (http://www.actuarialbookstore.com) Exam MLC seminar at Illinois State University: http://math.illinoisstate.edu/actuary/exams/prep_courses.shtml November 2012 Course MLC Examination, Problem No. 1 For two lives, (80) and (90), with independent future lifetimes, you are given: p80+ k p90+ k k 0 0.9 0.6 1 0.8 0.5 2 0.7 0.4 Calculate the probability that the last survivor will die in the third year. A. 0.20 B. 0.21 C. 0.22 D. 0.23 E. 0.24

Solution. The probability sought is = 2 p80:90 3 p80:90

( 2 p80 + 2 p90 2 p80:90 ) ( 3 p80 + 3 p90 3 p80:90 ) = = ( 2 p80 + 2 p90 2 p80 2 p90 ) ( 3 p80 + 3 p90 3 p80 3 p90 ) = = ( p80 p81 + p90 p91 p80 p81 p90 p91 ) ( p80 p81 p82 + p90 p91 p92 p80 p81 p82 p90 p91 p92 ) =
Using Poohsticks

= ( 0.9 0.8 + 0.6 0.5 0.9 0.8 0.6 0.5 )

= ( 0.72 + 0.3 0.216 ) ( 0.504 + 0.12 0.06048 ) = 0.24048. Answer E.

( 0.9 0.8 0.7 + 0.6 0.5 0.4 0.9 0.8 0.7 0.6 0.5 0.4 ) =

November 2012 Course MLC Examination, Problem No. 2 You are given: (i) An excerpt from a select and ultimate life table with a select period of 3 years: x x+3 l[ x ] l[ x ]+1 l[ x ]+2 lx+ 3 60 80,000 79,000 77,000 74,000 61 78,000 76,000 73,000 70,000 62 75,000 72,000 69,000 67,000 63 71,000 68,000 66,000 65,000 (ii) Deaths follow a constant force of mortality over each year of age. 63 64 65 66

Copyright 2012 by Krzysztof Ostaszewski. All rights reserved. No reproduction in any form is permitted without explicit permission of the copyright owner.

Calculate 1000 2 3 q[ 60 ]+0.75 . A. 104 B. 117 C. 122 D. 135 E. 142

Solution. Under constant force over each year of age, for 0 k 1, and x an integer,
k l lx+ k = k px = e kx = ( e x ) = x+1 , lx lx k

so that

lx+ k = (lx ) We have

1 k

(lx+1 )k .
l[ 60 ]+2.75 l[ 60 ]+5.75 l[ 60 ]+0.75
0.25 65 0.75 l66 0.75 0.75 63

1000 2 3 q[ 60 ]+0.75 = 1000


0.25 60 + 2

= 1000

l([ 60 ]+2 )+0.75 l65.75 l[ 60 ]+0.75

(l[ ] ) l l = 1000 (l[ ] ) (l[ ] )


0.25 60 60 +1

= 1000
Answer B.

77000

0.25

74000 0.75 67000 0.25 65000 0.75 116.70. 80000 0.25 79000 0.75

November 2012 Course MLC Examination, Problem No. 3 You are given:

t 4 (i) S0 ( t ) = 1 , for 0 t . 1 (ii) 65 = . 180 Calculate e106 , the curtate expectation of life at age 106.
A. 2.2 Solution. We have
1 d d t 4 t = ( ln S0 ( t )) = ln 1 = dt dt

B. 2.5

C. 2.7

D. 3.0

E. 3.2

=
Therefore,

1 d t 1 1 1 1 1 ln 1 = = . 4 dt 4 1 t 4 t

Copyright 2012 by Krzysztof Ostaszewski. All rights reserved. No reproduction in any form is permitted without explicit permission of the copyright owner.

1 1 1 = 65 = , 180 4 65

so that
1 1 = , 45 65 and = 110. Based on this
S (106 + t ) = 0 = S0 (106 ) 106 + t 4 1 110 106 4 1 110
1 1

p106

4t4 = . 4

Therefore,
e106 4 1 4 4 2 4 4 3 4 = 1 p106 + 2 p106 + 3 p106 + + + 2.4786. 4 p106 + ... = 4 4 4 =0
1 1 1

Answer B. November 2012 Course MLC Examination, Problem No. 4 For a special fully discrete whole life insurance on (40), you are given: (i) The death benefit is 50,000 in the first 20 years and 100,000 thereafter. (ii) Level benefit premiums of 1116 are payable for 20 years. (iii) Mortality follows the Illustrative Life Table. (iv) i = 0.06. Calculate 10V , the benefit reserve at the end of year 10 for this insurance. A. 13,340 B. 13,370 C. 13,400 D. 13,430 E. 13,460

Solution. Prospectively, we calculate the reserve as 1 50:10 = 10V = 50,000 A50:10 + 100,000 10 A50 1116 a

50 10 E50 a 60 ) = = 50,000 ( A50 + 10 E50 A60 ) 1116 ( a 13, 428.

= 50,000 ( 0.24905 + 0.51081 0.36913) 1116 (13.2668 0.5108 11.1451) =


Answer D. November 2012 Course MLC Examination, Problem No. 5 A special fully discrete 2-year endowment insurance with a maturity value of 2000 is issued to (x). The death benefit is 2000 plus the benefit reserve at the end of the year of death. For year 2, the benefit reserve is the benefit reserve just before the maturity benefit is paid. You are given: (i) i = 0.10.
Copyright 2012 by Krzysztof Ostaszewski. All rights reserved. No reproduction in any form is permitted without explicit permission of the copyright owner.

(ii) qx = 0.150 and qx+1 = 0.165. Calculate the level annual benefit premium. A. 1070 B. 1110 C. 1150 D. 1190 E. 1230

Solution. The initial reserve at time 0 is 0V = 0. The terminal reserve at time 2 is 2V = 2000, as the policy endows at that time. Let us write P for the level annual benefit premium sought. We use the recursive reserve formula twice to find it: ( 0V + P )(1 + i ) = qx ( 2000 + 1V ) + (1 qx ) 1V ,

( 1V + P )(1 + i ) = qx+1 ( 2000 + 2V ) + (1 qx+1 ) 2V .

Substituting known values, we obtain ( 0 + P ) 1.1 = 0.150 ( 2000 + 1V ) + 0.850 1V ,

From the first equation, 1V = 1.1P 300, and after we put this in the second equation (1.1P 300 + P ) 1.1 = 0.165 4000 + 0.835 2000, or 2660 P= 1151.52. 2.31 Answer C. November 2012 Course MLC Examination, Problem No. 6 For a special fully continuous 10-year increasing term insurance, you are given: (i) The death benefit is payable at the moment of death and increases linearly from 10,000 to 110,000. (ii) = 0.01. (iii) = 0.05. (iv) The annual premium rate is 450. (v) Premium-related expenses equal 2% of premium, incurred continuously. (vi) Claims-related expenses equal 200 at the moment of death. (vii) tV denotes the gross premium reserve at time t for this insurance. (viii) You estimate 9.6V using Eulers method with step size 0.2 and the derivative of tV at time 9.6. (ix) Your estimate of 9.8V is 126.68. Calculate the estimate of 9.6V . A. 230 B. 250 C. 270 D. 280 E. 290

( 1V + P ) 1.1 = 0.165 ( 2000 + 2000 ) + 0.835 2000.

Solution. Using Eulers method with step size 0.2 and the derivative of tV at time 9.6, we obtain

Copyright 2012 by Krzysztof Ostaszewski. All rights reserved. No reproduction in any form is permitted without explicit permission of the copyright owner.

d tV . dt t =9.6 Recall the Thieles differential equation when expenses are included: d V = t tV + Pt et ( St + Et tV ) x +t . dt t In this case, t = 9.6, 9.6 = = 0.05, tV is denoted by 9.6V and it is unknown, Pt is a gross
9.6

V 9.8V 0.2

premium with the annual rate of 450, et are premium related expenses at 2% of premium,
St is constant with S9.6 = 106,000, E9.6 = 200, so that

d tV = 0.05 9.6V + 450 0.02 450 (106,000 + 200 9.6V ) 0.01 = 621 + 0.06 9.6V . dt t =9.6 Substituting into the Eulers formula, we obtain d tV = 126.68 0.2 ( 612 + 0.06 9.6V ) = 250.88 0.012 9.6V . 9.6V 9.8V 0.2 dt t =9.6 This results in 250.88 247.91. 9.6V 1.012 Answer B.
November 2012 Course MLC Examination, Problem No. 7 You are calculating asset shares for a universal life insurance policy with a death benefit of 1000 on (x) with death benefits payable at the end of the year of death. You are given: (i) The account value at the end of year 4 is 30. (ii) The asset share at the end of year 4 is 20. (iii) At the beginning of year 5: a. A premium of 20 is paid. b. Annual cost of insurance charges of 2 and annual expense charges of 7 are deducted from the account value. (iv) At the beginning of year 5, the insurer incurs expenses of 2. (v) All withdrawals occur at the end of the policy year; the withdrawal benefit is the account value less a surrender charge of 20. (d ) (w) (vi) qx + 4 = 0.001 and q x + 4 = 0.050 are the probabilities of death and withdrawal, respectively. (vii) The annual interest rate for asset shares is 0.08; the annual interest rate credited to the universal life insurance policy is 0.06. Calculate the asset share at the end of year 5. A. 39 B. 40 C. 41 D. 42 E. 43

Solution. The account value (a.k.a., reserve) at the end of year 5 is


Copyright 2012 by Krzysztof Ostaszewski. All rights reserved. No reproduction in any form is permitted without explicit permission of the copyright owner.

30 + 20 9 1.06 43.46. Cost of insurance of 2 Accumulated at value at Premium Account plus annual expense the interest rate the end of year 4 credited charge of 7 (i.e., 6%) The surrender value at the end of year 5 is therefore this account value of 43.46 reduced by surrender charge of 20, i.e., 43.46 20 = 23.46. The asset share at the end of year 5 is Expected payout Accumulated AS4 Premium Expenses at withdrawal Expected death benefit payout incurred at asset share rate 20 + 20 2 1.08 0.001 1000 0.05 23.46 40.96. 1 0.001 0.05
Probability of policy continuing

Answer C. November 2012 Course MLC Examination, Problem No. 8 For a universal life insurance policy with a death benefit of 50,000 plus the account value, you are given: (i) Policy Year Monthly Percent of Monthly Monthly Surrender Premium Premium Cost of Expense Charge Charge Insurance Charge Rate per 1000 1 300 W% 2 10 500 2 300 15% 3 10 125
12 (ii) The credited interest rate is i ( ) = 0.054. (iii) The cash surrender value at the end of month 11 is 1200.00. (iv) The cash surrender value at the end of month 13 is 1802.94. Calculate W%, the percent of premium charge in policy year 1.

A. 25%

B. 30%

C. 35%

D. 40%

E. 45%

Solution. Let us write CSVk for the cash surrender value at time k, with time counted in months, and SCk for the surrender charge at time k. Let be COI k be the cost of insurance for month k. Since i (12 ) = 0.054, the effective monthly interest rate is

i (12 ) 0.054 j= = = 0.0045 = 0.45%. 12 12 Note that the cash surrender value equals the account value (i.e., reserve) minus the surrender charge. We are given that CSV13 = 1802.94. Based on this, 1802.94 = CSV13 = AV13 SC13 = AV13 125, so that
Copyright 2012 by Krzysztof Ostaszewski. All rights reserved. No reproduction in any form is permitted without explicit permission of the copyright owner.

AV13 = 1802.94 + 125 = 1927.94. Similarly, AV11 = CSV11 + SC11 = 1200 + 500 = 1700. Now we use the recursive formula for the account value (reserve) for universal life with increasing total benefit and fixed ADB AVt 1 + Gt (1 rt ) et (1 + it ) = qx +t ( B + AVt +1 ) + px +t AVt = qx +t B + AVt .

The cost of insurance in this formula is COI t = qx+t B, so that


AVt = AVt 1 + Gt (1 rt ) et (1 + it ) COI t .

In this case

AV13 = AV12 + G13 (1 r13 ) e13 (1 + i13 ) COI13 ,


11

AV12

( = ( AV ( (

+ G12 (1 r12 ) e12

) )(1 + i ) COI
12

12

Substituting known values (note that the cost of insurance figures are given per thousand), we obtain 1927.94 = AV12 + 300 (1 0.15) 10 1.0045 50 3,
AV12 = 1700 + 300 (1 W ) 10 1.0045 50 2.

From the first equation 1927.94 + 50 3 AV12 = + 10 300 (1 0.15) 1823.63116, 1.0045 and substituting this into the second equation, we obtain 1823.63116 + 50 2 1700 + 10 1.0045 W 1 0.25. 300 Answer A. November 2012 Course MLC Examination, Problem No. 9 You are given the following about a universal life insurance policy on (60): (i) The death benefit equals the account value plus 200,000. (ii) Age x Annual Premium Annual Cost of Annual Expense Insurance Rate per Charges 1000 60 5000 5.40 100 61 5000 6.00 100 (iii) Interest is credited at 6%. (iv) Surrender value equals 93% of account value during the first two years. Surrenders occur at the ends of policy years. (v) Surrenders are 6% per year of those who survive. (vi) Mortality rates are 1000 q60 = 3.40 and 1000 q61 = 3.80. (vii) i = 7%.
Copyright 2012 by Krzysztof Ostaszewski. All rights reserved. No reproduction in any form is permitted without explicit permission of the copyright owner.

Calculate the present value at issue of the insurers expected surrender benefits paid in the second year. A. 380 B. 390 C. 400 D. 410 E. 420

Solution. The account value at time 0, just before the first premium is paid is zero. Then, since death benefit equals fixed ADB plus account value,

AV1 = AV0 + G1 (1 r1 ) e1 (1 + i1 ) q60 B = ( 0 + 5000 1 100 ) 1.06 200 5.40 = 4114.

Following into the second year, AV2 = AV1 + G2 (1 r2 ) e2 (1 + i2 ) q61 B =

= ( 4114 + 5000 1 100 ) 1.06 200 6.00 8354.84.

If a policy is surrendered in year 2 (and for a policy in existence at the end of year 1 this happens with probability 0.06), then it was not surrendered in year 1 (and for a policy issued at time 0 this happens with probability 1 0.06 = 0.94), so that the present value at time 0 of the expected surrender benefits in year 2 is 2 8354.84 1 0.06 ) (1 0.0034 ) (1 0.0038) 1.07 0.06 ( 0.93 Present value Adjusted for Fraction that Account value
available for surrenders surrender charge surrenders in Fraction that did not surrender year 2 in year 1 1 q60 1 q61 factor

380.01.
Answer A. November 2012 Course MLC Examination, Problem No. 10 For a special 3-year term life insurance policy on (x) and (y) with dependent future lifetimes, you are given: (i) A death benefit of 100,000 is paid at the end of the year of death if both (x) and (y) die within the same year. No death benefits are payable otherwise. (ii) px+ k = 0.84366, k = 0, 1, 2. (iii) py+ k = 0.86936, k = 0, 1, 2. (iv) px+ k:y+ k = 0.77105, k = 0, 1, 2. (v) Maturity (in years) Annual Effective Spot Rate 1 3% 2 8% 3 10% Calculate the expected present value of the death benefit. A. 9,500 B. 10,100 C. 12,100 D. 12,500 E. 14,100

Solution. The death benefit of $100,000 is paid at the end of year 1 if both (x) and (y) die during the first year, or it is paid at the end of year 2 if both (x) and (y) are alive at time 1 and
Copyright 2012 by Krzysztof Ostaszewski. All rights reserved. No reproduction in any form is permitted without explicit permission of the copyright owner.

both die during the second year, or it is paid at the end of year 3 if both (x) and (y) are alive at time 2 and both die during the third year. Let us note the following, true for k = 0, 1, or 2: qx+ k:y+ k = 1 px+ k:y+ k = 1 px+ k + py+ k px+ k:y+ k =
= 1 ( 0.84366 + 0.86936 0.77105 ) = 0.05803.
Poohsticks

Also,
px:y = px:y px +1:y+1 = 0.77105 0.77105 = 0.77105 2. The expected present value we are looking for is therefore 1 2 100,000 1.03 qx:y + 100,000 1.08 px:y qx+1:y+1 + Present value of death benefit Present value of death benefit
2

payment made at time 1

Probability that both ( x ) and ( y ) die during first year

payment made at time 2

Probability that both ( x ) and ( y ) die during second year

3 + 100,000 1.10 2 p x:y q x + 2:y+ 2 = Present value of death benefit payment made at time 2 Probability that both ( x ) and ( y ) die during second year

0.05803 0.77105 0.05803 0.77105 2 0.05803 = 100,000 + + 12,062.09. 1.08 2 1.10 3 1.03

Answer C. November 2012 Course MLC Examination, Problem No. 11 For a whole life insurance of 1000 on (70), you are given: (i) Death benefits are payable at the end of the year of death. (ii) Mortality follows the Illustrative Life Table. (iii) Maturity (in years) Annual Effective Spot Rate 1 1.6% 2 2.6% (iv) For the year starting at time k 1 and ending at time k, k = 3, 4, 5, ... , the one-year forward rate is 6%. Calculate the expected present value of the death benefits. A. 520 B. 530 C. 550 D. 570 E. 600

Solution. It seems like it will be a lot of calculations to go from age 70 to the limiting age if we do every year calculation separately. But the Illustrative Life Table of course gives us actuarial present values of whole life insurance death benefits, except that it does so for the valuation interest rate of 6%. Here, the spot rates for the first two years are well below 6%, but beyond that all one-year forward rates are 6% (so that all rates for all periods starting at time 2 or later are also 6%). Thus starting from age 72, we can determine actuarial present values from the Illustrative Life Table. We have

Copyright 2012 by Krzysztof Ostaszewski. All rights reserved. No reproduction in any form is permitted without explicit permission of the copyright owner.

q p p q 1000 A70 = 1000 70 + p70 71 2 + 70 71 A72 , 2 1.016 1.026 1.026 where all values on the right-hand side can now be looked up in the Illustrative Life Table. From the table: q70 = 0.03318, q71 = 0.03626, A72 = 0.54560, and therefore p70 = 0.96682, p71 = 0.96374, so that 0.03626 0.96682 0.96374 0.03318 1000 A70 = 1000 + 0.96682 + 0.54560 2 2 1.016 1.026 1.026
548.89.
Answer C. November 2012 Course MLC Examination, Problem No. 12 A party of scientists arrives at a remote island. Unknown to them, a hungry tyrannosaur lives on the island. You model the future lifetimes of the scientists as a three-state model, where: State 0: no scientists have been eaten. State 1: exactly one scientist has been eaten. State 2: at least two scientists have been eaten. You are given: (i) Until a scientist is eaten, they suspect nothing, so t01 = 0.01 + 0.02 2 t , t > 0. (ii) Until a scientist is eaten, they suspect nothing, so the tyrannosaur may come across two together and eat both, with t02 = 0.5 t01 , t > 0. (iii) After the first death, scientists become much more careful, so t12 = 0.01, t > 0. Calculate the probability that no scientists are eaten in the first year. A. 0.928 B. 0.943 C. 0.951 D. 0.956 E. 0.962

Solution. Note that because it is impossible to return to state 0, the probability we are looking for, 00 00 1 p0 is the same as 1 p0 . We calculate
00 00 1 p0 = 1 p0 = e

( t
0

01

+ t02 dt

=e

( t
0

01

+ 0.5 t01 dt

=e

1.5 0.01+ 0.022t dt


0

ln 2 ln 2 ln 2 =e =e =e 0.94338496. We could also note that t01 + t02 = 0.01 + 0.02 2 t + 0.5 0.01 + 0.02 2 t = 0.015 + 0.03 2 t ,

0.03 t 0.015 t + 2 ln 2 t = 0

t =1

0.06 0.03 0.015 + 0

0.015

0.03

and this is the Makehams Law with A = 0.015, B = 0.03, c = 2. Recall that under Makehams Law, n px = s n g
c x c n 1

) , where g = e ln c and s = e A . Here, g = e ln 2 ,


B 0.03

Copyright 2012 by Krzysztof Ostaszewski. All rights reserved. No reproduction in any form is permitted without explicit permission of the copyright owner.

0.03 00 00 , so that the quantity sought, i.e., 1 p0 = 1 p0 , s = e0.015 , and n px = e0.015 n e ln 2 is the same as 1 p0 under this Makeham model, i.e., (211) 0.03 00 00 0.0151 e ln 2 0.94338496. 1 p0 = 1 p0 = e Answer B.
November 2012 Course MLC Examination, Problem No. 13 You are given: (i) The following excerpt from a triple decrement table: x l ( ) d (1) d (2)
x x x

2 x 2 n 1

( 3) dx 50 100,000 490 8,045 1,100 51 90,365 -8,200 -52 80,000 ---(ii) All decrements are uniformly distributed over each year of age in the triple decrement table. ( 3) (iii) q is the same for all ages. x

Calculate 10,000 q51 (1) . A. 130 B. 133

C. 136

D. 138

E. 141

Solution. Recall the key formula for UDD over each year of age in the multiple decrement table:
p x
( j)

= px

( )
( )

) q( x

j) q( x

We have

p50

( )

( ) l51 90, 365 = ( ) = = 0.90365, l50 100,000 ( 3) d50 1,100 = 0.011, ( ) = l50 100,000

( 3) q50 =

( ) ( ) q50 = 1 p50 = 1 0.90365 = 0.09635. We calculate therefore

q51 = q50 = 1 p50 = 1 p50

( 3)

( 3)

( 3)

This means that p51 ( 3) 0.9885 and


( ) 0.9855 p51 ( 3) = p51

( )
( )

( ) q50

( ) q50
3

= 1 0.90365 0.09635 0.0115.


( ) q51 80,000 1 90,365
3

0.011

( )

( ) q51

( ) q51
3

80,000 = 90, 365

From this we calculate


Copyright 2012 by Krzysztof Ostaszewski. All rights reserved. No reproduction in any form is permitted without explicit permission of the copyright owner.

80,000 1 ln 0.9855 90, 365 ( 3) q51 0.010889815. 80,000 ln 90, 365 This means that ( 3) ( 3) ( ) d51 = q51 l51 0.010889815 90, 365 984.0581,
(1) ( ) (2) ( 3) ( ) ( ) (2) ( 3) d51 = d51 d51 d51 = l50 l51 d51 d51

( 90, 365 80,000 ) 8200 984 = 1,181,


(1) d51 1,181 q51 = ( ) 0.01306922, l51 90, 365 and we also have 80,000 ( ) p51 = 0.88529851, 90, 365 80,000 ( ) ( ) q51 = 1 p51 = 1 0.11470149. 90, 365 We therefore conclude that (1)

p51 = p51

(1)

( )
( )

( ) q51

() q51
1

0.885298510.11470149 0.98621441.

0.01306922

The quantity sought is Answer D.

10,000 q51 (1) = 10,000 1 p51 (1) 10,000 (1 0.98621441) 138.

November 2012 Course MLC Examination, Problem No. 14 For a special whole life insurance policy issued on (40), you are given: (i) Death benefits are payable at the end of the year of death. (ii) The amount of benefit is 2 if death occurs within the first 20 years and is 1 thereafter. (iii) Z is the present value random variable for the payments under this insurance. (iv) i = 0.03. (v) x Ax 20 E x 40 0.36987 0.51276 60 0.62567 0.17878 (vi) E Z 2 = 0.24954. Calculate the standard deviation of Z. A. 0.27 B. 0.32 C. 0.37 D. 0.42 E. 0.47

( )

Solution. The present value of death benefit random variable is


Copyright 2012 by Krzysztof Ostaszewski. All rights reserved. No reproduction in any form is permitted without explicit permission of the copyright owner.

K +1 2 v , K < 20, Z = K +1 v , K 20. Therefore, E ( Z ) = 2 A40 20 E40 A60 = 2 0.36987 0.51276 0.62567 0.41892. We conclude that the variance of Z is

Var ( Z ) = E ( Z 2 ) ( E ( Z )) 0.24954 0.41892 2 0.07405,


2

and the standard deviation of Z is

Z = Var ( Z ) 0.07405 0.27212. Answer A.


November 2012 Course MLC Examination, Problem No. 15 For a special 2-year term insurance policy on (x), you are given: (i) Death benefits are payable at the end of the half-year of death. (ii) The amount of the death benefit is 300,000 for the first half-year and increases by 30,000 per half-year thereafter. (iii) qx = 0.16 and qx+1 = 0.23. (iv) i ( ) = 0.18. (v) Deaths are assumed to follow a constant force of mortality between integral ages. (vi) Z is the present value random variable for this insurance. Calculate Pr(Z > 277,000).
2

A. 0.08

B. 0.11

C. 0.14

D. 0.18

E. 0.21

Solution. Note that the effective half a year interest rate is 9%. Also, under the constant force of mortality between integral ages assumption, if we write x for the constant force between ages x and x + 1, and x +1 for the constant force between ages x +1 and x + 2, then 0.5 x 0.5 = 1 px = 1 1 0.16 0.083485, 0.5 q x = 1 0.5 p x = 1 e
0.5 0.5 0.5 0.5 qx+0.5 = 1 0.5 px+0.5 = 1 px = 1 1 0.16 0.083485, 0.5 qx+1 = 1 0.5 px+1 = 1 e0.5 x+1 = 1 px +1 = 1 1 0.23 0.122504,

0.5 qx+1.5 = 1 0.5 px+1.5 = 1 e0.5 x+1 = 1 px +1 = 1 1 0.23 0.122504. Based on the above, probabilities of the insured dying in each of the four half-years are: q = 0.5 qx 0.083485, 0 0.5 x
0.5 0.5 1 0.5

qx =

0.5

qx = 1 px 0.5 px +1 0.5 qx +1.5 (1 0.16 ) 0.877496 0.122504 0.090297. The random present value of the death benefit is
1 0.5

qx = 1 px 0.5 qx +1 (1 0.16 ) 0.122504 0.102903,

px 0.5 qx +0.5 0.916515 0.083485 0.076515,

Copyright 2012 by Krzysztof Ostaszewski. All rights reserved. No reproduction in any form is permitted without explicit permission of the copyright owner.

300,000 275,229.36 with probability 0.5 qx 0.083485, 1.9 330,000 277, 754.40 with probability 0.5 0.5 qx 0.076515, 1.9 2 Z= 360,000 277,986.05 with probability q 0.102903, 1 0.5 x 1.9 3 390,000 276,285.83 with probability 1 0.5 qx 0.090297. 4 1.9 Also, Z = 0 otherwise. We conclude that Pr ( Z > 277,000 ) = 0.5 0.5 qx + 1 0.5 qx 0.076515 + 0.102903 0.179418. Answer D.
November 2012 Course Examination, Problem No. 16 multiple You are evaluating the financial strengthMLC of companies based on the following You are evaluating the financial strength of companies based on the following multiple state model: state model:

State 0 Solvent

State 1 Bankrupt

State 2 Liquidated
For each company, you assume the following constant transition intensities: 01 For each company, = assume 0.02. the following constant transition intensities: (i) you

(i) (ii) (iii)

10 = 0.06. (ii) 01 = 0.02 12 (iii) = 0.10.

1 10 = 0.06 Using Kolmogorovs forward equations with step h = , calculate the probability that a 2 12 company currently Bankrupt will be Solvent at the end of one year. = 0.10

A. 0.048

B. 0.051

C. 0.054

D. 0.057

E. 0.060

Using Kolmogorovs forward equations with step h = 1/ 2, calculate the probability that a Solution. company currently Bankrupt will Solvent at the end of one year. We have, using thebe Markov process notation: (A) (B) (C) (D) 0.048 0.051 0.054 0.057

02 ( t ) = 02 = 02 = 0.00, 0 ( t ) = 0 = 01 + 02 = 0.02 + 0.00 = 0.02,


Copyright 2012 by Krzysztof Ostaszewski. All rights reserved. No reproduction in any form is permitted without explicit permission of the copyright owner.

01 ( t ) = 01 = 01 = 0.02,

12 ( t ) = 12 = 12 = 0.10, 1 ( t ) = 1 = 10 + 12 = 0.06 + 0.10 = 0.16, 21 ( t ) = 21 = 21 = 0.00, 2 ( t ) = 2 = 20 + 21 = 0 + 0 = 0. This tells us that the generator matrix is: (t + t ) (t + r ) (t + r ) 0 01 02 0 0.02 0.02 Qt +r 10 ( t + r ) 1 ( t + t ) 12 ( t + r ) = 0.06 0.16 0.1 . 0 0 0 t + r t + r t + t ( ) ( ) ( ) 21 2 20 For p(t ) p(t ) p(t ) r 00 r 01 r 02 (t ) (t ) (t ) , r p10 rP t = r p11 r p12 (t ) (t ) (t ) r p20 r p21 r p22 the resulting Kolmogorov Forward Equation, in matrix form, is d d d (t ) (t ) (t ) r p00 r p01 r p02 (t ) (t ) p(t ) dr dr dr r 00 r p01 r p02 d d d d (t ) (t ) (t ) (t ) (t ) (t ) = d r p10 rP t = r p10 r p11 r p12 r p11 r p12 dr dr dr dr dr (t ) (t ) (t ) r p20 p p r 21 r 22 d p(t ) d p(t ) d p(t ) dr r 20 dr r 21 dr r 22
=
r r r

10 ( t ) = 10 = 10 = 0.06,

20 ( t ) = 20 = 20 = 0.00,

(t ) p00 (t ) p10 (t ) p20

r r r

(t ) p01 (t ) p11 (t ) p21

(t ) p02 0 0.02 0.02 (t ) 0.06 0.16 0.1 = r p12 (t ) 0 0 0 p r 22


r

0.02 p(t ) + 0.06 p(t ) 0.02 p(t ) 0.16 p(t ) 0.1 p(t ) r 00 r 01 r 00 r 01 r 01 t t t t ( ) ( ) ( ) ( ) (t ) . = 0.02 r p10 + 0.06 r p11 0.02 r p10 0.16 r p11 0.1 r p11 (t ) (t ) (t ) (t ) (t ) 0.02 r p20 + 0.06 r p21 0.02 r p20 0.16 r p21 0.1 r p21 (0) We are looking for an approximate value of the probability 1 p10 . We can actually remove the superscript related to the starting point in time, because due to constant intensities of (0) transition, the starting time does not affect probabilities, so 1 p10 = 1 p10 , and similarly we will drop the superscript for other probabilities. The equation d (t ) (t ) (t ) r p10 = 0.02 r p10 + 0.06 r p11 dr
Copyright 2012 by Krzysztof Ostaszewski. All rights reserved. No reproduction in any form is permitted without explicit permission of the copyright owner.

gives us this approximation 1 p10 0.5 p10 0.02 0.5 p10 + 0.06 0.5 p11 , 0.5 or 1 p10 0.99 0.5 p10 + 0.03 0.5 p11 . Additionally d d (t ) (t ) (t ) (t ) (t ) (t ) r p10 = 0.02 r p10 + 0.06 r p11 r p11 = 0.02 r p10 0.16 r p11 dr dr give us approximations 0.5 p10 0 p10 0.5 p11 0 p11 0.02 0 p10 + 0.06 0 p11 , 0.02 0 p10 0.16 0 p11 , 0.5 0.5 or 0.5 p10 0.99 0 p10 + 0.03 0 p11 , 0.5 p11 0.01 0 p10 + 0.92 0 p11 . But 0 p10 = 0, 0 p11 = 1, so that 0.5 p10 0.99 0 p10 + 0.03 0 p11 = 0.99 0 + 0.03 1 = 0.03, 0.5 p11 0.01 0 p10 + 0.92 0 p11 = 0.01 0 + 0.92 1 = 0.92. We conclude that 1 p10 0.99 0.5 p10 + 0.03 0.5 p11 0.99 0.03 + 0.03 0.92 0.0573. Answer D. November 2012 Course MLC Examination, Problem No. 17 For a whole life insurance of 1000 with semi-annual premiums on (80), you are given: (i) A gross premium of 60 is payable every 6 months starting at age 80. (ii) Commissions of 10% are paid each time a premium is paid. (iii) Death benefits are paid at the end of the quarter of death. (iv) tV denotes the gross premium reserve at time t. (v) 10.75V = 753.72. (vi) t l90+t 0 1000 0.25 898 0.50 800 0.75 706 (vii) i ( ) = 0.08. Calculate 10.25V .
4

A. 680

B. 690

C. 700

D. 710

E. 730

Solution. We can use the recursive reserve formula twice, in quarterly steps. Note that the effective quarterly interest rate is 2%, and that when going from policy duration 10.25 to 10.75, we only have a premium payment at policy duration 10.50. Thus
Copyright 2012 by Krzysztof Ostaszewski. All rights reserved. No reproduction in any form is permitted without explicit permission of the copyright owner.

( 10.25V + 0 ) 1.02 = 0.25 q90.25 1000 + 0.25 p90.25 10.50V , ( V + 60 (1 0.10 )) 1.02 = q 1000 + p
10.50 0.25 90.50 0.25

90.50

10.75V .

Now we substitute the following known data: 10.75V = 753.72, l l 898 800 98 0.25 d90.25 = 90.25 90.50 = = , 0.25 q90.25 = l90.25 l90.25 898 898 l90.50 800 = , 0.25 p90.25 = l90.25 898 l l 800 706 94 0.25 d90.50 = 90.50 90.75 = = , 0.25 q90.50 = l90.50 l90.50 800 800 l90.75 706 = , 0.25 p90.50 = l90.50 800 we obtain 98 800 1000 + 10.50V , 10.25V 1.02 = 898 898 94 706 ( 10.50V + 54 ) 1.02 = 800 1000 + 800 753.72. From the second equation 94 706 1000 + 753.72 800 800 V = 54 753.3117 10.50 1.02 Using this in the first equation, we obtain 98 800 1000 + 753.3117 898 898 V 729.9984. 10.25 1.02 Answer E. November 2012 Course MLC Examination, Problem No. 18 For a special fully discrete whole life insurance on (40), you are given: (i) The death benefit is 1000 during the first 11 years and 5000 thereafter. (ii) Expenses, payable at the beginning of the year, are 100 in year 1 and 10 in years 2 and later. (iii) is the level annual premium, determined using the equivalence principle. (iv) G = 1.02 is the level annual gross premium. (v) Mortality follows the Illustrative Life Table. (vi) i = 0.06. (vii) 11 E40 = 0.50330. Calculate the gross premium reserve at the end of year 1 for this insurance. A. 70 B. 60 C. 50 D. 40 E. 30

Copyright 2012 by Krzysztof Ostaszewski. All rights reserved. No reproduction in any form is permitted without explicit permission of the copyright owner.

Solution. We begin by finding . The actuarial present value of benefits at issue is 1000 A40 + 4 11 E40 1000 A51 = 161.32 + 4 0.50330 259.61 683.9669. The actuarial present value of expenses at issue is 40 1) = 100 + 10 (14.8166 1) = 238.17. 100 + 10 a40 = 100 + 10 ( a All the values used in calculations above came from the Illustrative Life Table. The premium is paid as a life annuity due on (40), hence 683.97 + 238.17 683.97 + 238.17 = 62.2365. 40 a 14.8166 Therefore, G = 1.02 63.4812. If we calculated the reserve retrospectively, we would obtain G 1000 q40 1.06 1 100 63.4812 2.78 1.06 1 100 41.6056. (1 0.00278 ) 1.06 1 1 E 40 But prospectively, we obtain 41 63.4812 a 41 = 1000 A41 + 4 10 E41 1000 A51 ) + 10 a (
Actuarial present value of future benefits Actuarial present value of future expenses Actuarial present value of future gross premiums

= 168.69 + 4 0.53499 259.61 + 10 14.6864 63.48 14.6864 61.2012. The difference of these two is ( 41.6056 ) ( 61.2012 ) 19.60. Note that the actuarial present value at policy inception of the profit portion of the gross premium is 40 0.02 62.2365 14.8166 18.4427. 0.02 a Its actuarial accumulated value at policy duration 1 is approximately 18.4427 18.4427 = 19.60. (1 0.00278 ) 1.06 1 1 E 40 Let us write the actuarial equivalence for gross premium valuation at policy inception: Actuarial Present Value of Future Gross Premiums = = Actuarial Present Value of Future Benefits and Expenses + + Actuarial Present Value of Future Profits. Note that at policy inception, the actuarial present value of future benefits and expenses minus the actuarial present value of future gross premiums equals the opposite (i.e., minus) of the actuarial present value of future profits. Now consider a policy duration k, where k is an integer. Let us write APV for actuarial present value, and AAV for actuarial accumulated value. We have at policy inception: APV (Future Gross Premiums) = = APV (Future Gross Premiums up to time k) + + APV (Future Gross Premiums from time k on) = APV (Future Benefits and Expenses) + APV (Future Profits) = = APV (Future Benefits and Expenses up to time k) + + APV (Future Benefits and Expenses from time k on) + + APV (Future Profits). Now we accumulate all of these amounts to policy duration k
Copyright 2012 by Krzysztof Ostaszewski. All rights reserved. No reproduction in any form is permitted without explicit permission of the copyright owner.

AAV (Past Gross Premiums up to time k) + + APV (Future Gross Premiums from time k on) = = AAV (Past Benefits and Expenses up to time k) + + APV (Future Benefits and Expenses from time k on) + + Actuarial Value (Past and Future Profits). Therefore, at policy duration k, Retrospective Reserve = AAV (Past Gross Premiums) AAV (Past Benefits and Expenses) = = APV (Future Benefits and Expenses) APV (Future Gross Premiums) + + Actuarial Value (Past and Future Profits) = = Prospective Reserve + Actuarial Value (Past and Future Profits). If you recall that the purpose of the reserve is to provide funds for benefits and expenses payment that are not paid by remaining premiums, you now should be able to clearly see that the prospective reserve calculation is the correct one, and if you do the calculation retrospectively, you must subtraction the actuarial value of all profits at the moment of calculation from the retrospective calculation result. In this problem, the reserve at policy duration 1 is: 61.20 = 41.60 19.60. Interestingly, 40 was one of the answer choices, nice little trap waiting for the candidates taking the exam. The correct answer choice is 60, the one closest to 61.20. Answer B. November 2012 Course MLC Examination, Problem No. 19 For whole life annuities-due of 15 per month on each of 200 lives age 62 with independent future lifetimes, you are given: (i) i = 0.06. (12 ) (12 ) (ii) A62 = 0.4075 and 2 A62 = 0.2105. (iii) is the single premium to be paid by each of the 200 lives. (iv) S is the present value random variable at time 0 of total payments made to the 200 lives. Using the normal approximation, calculate such that Pr(200 > S) = 0.90. A. 1850 B. 1860 C. 1870 D. 1880 E. 1890

Solution. Let X1 , X2 , , X 200 , be the random present values of life annuities-due considered. These are independent identically distributed random variables, and S = X1 + X2 + ... + X200 . This implies that S is approximately normal and E ( S ) = E ( X1 + X2 + ... + X200 ) = 200 E ( X1 ) as well as Var ( S ) = Var ( X1 + X2 + ... + X200 ) = 200Var ( X1 ) .
1 Recall that d (12 ) = 12 1 (1 + i ) 12 . We have

Copyright 2012 by Krzysztof Ostaszewski. All rights reserved. No reproduction in any form is permitted without explicit permission of the copyright owner.

(12 ) 62 E ( X1 ) = 15 12 a = 15 12

(12 ) 1 A62 = 15 12 d (12 )

1 0.4075 1834.7545, 1 12 1 1.06 12

and

Var ( X1 ) = (15 12 )
2

(12 ) (12 ) A62 A62

(d( ) )
12

= 180
2

0.2105 0.4075 2
1 12 1 1.06 12 2

426,176.9086.

Based on this, E ( S ) = 200 E ( X1 ) 200 1834.7545 366,936.12, and Var ( S ) = 200Var ( X1 ) 200 426,176.9086 85,235, 381.71. We have 200 E ( S ) S E ( S ) 0.90 = Pr ( 200 > S ) = Pr > . Var ( S ) Var ( S ) S E (S ) 200 E ( S ) Since is approximately standard normal, must be the 90-th Var ( S ) Var ( S ) percentile of the standard normal distribution, i.e., 1.282. This gives the equation 200 E ( S ) 200 366,936.12 1.282 = = , 85,235, 381.71 Var ( S ) resulting in

=
Answer E.

366,936.12 + 1.282 85,235, 381.71 1893.8597. 200

November 2012 Course MLC Examination, Problem No. 20 Stuart, now age 65, purchased a 20-year deferred whole life annuity-due of 1 per year at age 45. You are given: (i) Equal annual premiums, determined using the equivalence principle, were paid at the beginning of each year during the deferral period. (ii) Mortality at ages 65 and older follows the Illustrative Life Table. (iii) i = 0.06. (iv) Y is the present value random variable at age 65 for Stuarts annuity benefits. Calculate the probability that Y is less than the actuarial accumulated value of Stuarts premiums. A. 0.40 Solution.
Copyright 2012 by Krzysztof Ostaszewski. All rights reserved. No reproduction in any form is permitted without explicit permission of the copyright owner.

B. 0.42

C. 0.44

D. 0.46

E. 0.48

Let us write P for the annual premium that Stuart has paid. Since it was calculated using the equivalence principle, we have E a P = 20 45 65 . 45:20 a Let us write K 65 for the curtate future lifetime of (65) (we could also write K(65), but we need to get used to the new notation, so this is a good time to practice). The probability 65 = 9.8969 from the Illustrative Life Table) we are looking for is (note that a
Pa K +1 = Pr Pr 45:20 > a 65 20 E45
20

65 > a K = Pr a

65 a E45 a K +1 = 45:20 > a 65 45:20 a 20 E 45


65 +1

) = Pr (9.8969 > a ).
K 65 +1

If we use BA II Plus Pro, set BGN mode by pushing 2ND, BGN if BGN is not displayed, and enter 0 FV, 1 PMT, 9.8969 PV, 6 I/Y, CPT N, we obtain 14.0973895. This means that the probability sought is Pr ( K 65 + 1 < 14.0973895 ) = Pr ( K 65 + 1 14 ) = 1 14 p65 =

l79 4225163 = 1 0.43917133. l65 7533964 For a moment, when you see the formula for P you might worry about working with mortality from before age 65, when we are told that mortality follows the Illustrative Life Table only from age 65 on, but as you can see, items related to mortality from before age 65 cancel nicely in the work involved in finding the probability sought. Answer C. = 1
November 2012 Course MLC Examination, Problem No. 21 For a fully continuous whole life insurance issued on (x) and (y), you are given: (i) The death benefit of 100 is payable at the second death. (ii) Premiums are payable until the first death. (iii) The future lifetimes of (x) and (y) are dependent. 1 3 (iv) t pxy = e0.01t + e0.03t , t 0. 4 4 0.01t , t 0. (v) t px = e 0.02 t , t 0. (vi) t py = e (vii) = 0.05. Calculate the annual benefit premium rate for this insurance. A. 0.96 B. 1.43 C. 1.91 D. 2.39 E. 2.86

Solution. Note that the future lifetime of (x) is ruled by constant force of mortality of 1%, and the future lifetime of (y) is rules by constant force of mortality of 2%. Let us write P for the annual benefit premium rate sought. We have
Copyright 2012 by Krzysztof Ostaszewski. All rights reserved. No reproduction in any form is permitted without explicit permission of the copyright owner.

P=
Also,

100 Ax:y ax:y


+

.
3 1 e0.01t + e0.03t dt = 4 4 3 4
+

ax:y = =
Ax:y

e
0

0.05 t

1 4
=

0.06 t e dt + 0

e
0

0.08 t

dt =

1 1 3 1 + 13.5416667, 4 0.06 4 0.08

Poohsticks

Ax + Ay Ax:y = Ax + Ay 1 ax:y =

0.01 0.02 + (1 0.05 13.5416667 ) 0.12946429. 0.01 + 0.05 0.02 + 0.05

Therefore,

P=
Answer A.

100 Ax:y ax:y

100 0.12946429 0.95604396. 13.5416667

November 2012 Course MLC Examination, Problem No. 22 You are given the following information about a special fully discrete 2-payment, 2-year term insurance on (80): (i) Mortality follows the Illustrative Life Table. (ii) i = 0.0175. (iii) The death benefit is 1000 plus a return of all premiums paid without interest. (iv) Level premiums are calculated using the equivalence principle. Calculate the benefit premium for this special insurance. A. 82 B. 86 C. 90 D. 94 E. 98

Solution. Let us write P for the level annual premium sought. Based on the equivalence principle, we write
1 80:2 = 1000 A80:2 Pa + P ( IA )80:2 , 1

so that

80:2 ( IA )80:2 a We calculate


1

P=

1 1000 A80:2

.
1 q80 1 0.0803 = 1+ 1.90388206, 1.0175 1.0175

80:2 = 1 + vp80 = 1 + a

Copyright 2012 by Krzysztof Ostaszewski. All rights reserved. No reproduction in any form is permitted without explicit permission of the copyright owner.

p q q 1 1000 A80:2 = 1000 80 + 80 81 2 = 1.0175 1.0175 0.0803 0.9197 0.08764 = 1000 + 156.772702, 1.0175 1.0175 2
and

( IA )180:2
Therefore,

q80 2 p80 q81 0.0803 2 0.9197 0.08764 + = + 0.23462648. 1.0175 1.0175 2 1.0175 1.0175 2

P=
Answer D.

80:2 ( IA )80:2 a
1

1 1000 A80:2

156.772702 93.9177342. 1.90388206 0.23462648

November 2012 Course MLC Examination, Problem No. 23 A life insurance company issues fully discrete 20-year term insurance policies of 1000. You are given: (i) Expected mortality follows the Illustrative Life Table. (ii) Death is the only decrement. (iii) 3V , the reserve at the end of year 3, is 12.18. On January 1, 2009, the company sold 10,000 of these policies to lives all aged 45. You are also given: (i) During the first two years, there were 30 actual deaths from these policies. (ii) During 2011, there were 18 actual deaths from these policies. Calculate the companys gain due to mortality for the year 2011. A. 28,100 B. 28,300 C. 28,500 D. 28,700 E. 28,900

Solution. The mortality gain equals ( Expected deaths Actual deaths ) Net Amount at Risk. In this case, the net amount at risk is 1000 3V = 1000 12.18 = 987.82. Expected deaths are q47 from the Illustrative Life Table, i.e., 0.00466, times the number of alive policyholders at the beginning of 2011, which is 10,000 30 = 9970, which gives 0.00466 9970 46.4602. The actual number of deaths in 2011 is 18. Hence, the mortality gain equals ( 46.4602 18 ) 987.82 28,113.5548. Answer A. November 2012 Course MLC Examination, Problem No. 24 An insurance company is designing a special 2-year term insurance. Transitions are modeled as a four-state homogeneous Markov model with states:
Copyright 2012 by Krzysztof Ostaszewski. All rights reserved. No reproduction in any form is permitted without explicit permission of the copyright owner.

(H) Healthy (Z) infected with virus Zebra (L) infected with virus Lion (D) Death The annual transition probability matrix is given by: H Z L D H 0.90 0.05 0.04 0.01 Z 0.10 0.20 0.00 0.70 L 0.20 0.00 0.20 0.60 D 0.00 0.00 0.00 1.00 You are given: (i) Transitions occur only once per year. (ii) 250 is payable at the end of the year in which you become infected with either virus. (iii) For lives infected with either virus, 1000 is payable at the end of the year of death. (iv) The policy is issued only on healthy lives. (v) i = 0.05. Calculate the actuarial present value of the benefits at policy issue. A. 66 B. 75 C. 84 D. 93 E. 102

Solution. We list all possible cases of payouts, with their probabilities and actuarial present values (calculated as products of probabilities and discounted benefits) Possible Probability Discounted Actuarial transition benefit present value 250 H Z 0.05 11.9047619 1.05 250 H L 0.04 9.52380952 1.05 1000 0.05 0.7 H Z D 31.7460317 1.05 2 1000 0.04 0.6 H L D 21.7687075 1.05 2 250 0.9 0.05 H H Z 10.2040816 1.05 2 250 0.9 0.04 H H Z 8.16326531 1.05 2 The total actuarial present value of the benefits at policy issue is the sum of all numbers in the last column, which is approximately 93.3106576. Answer D. November 2012 Course MLC Examination, Problem No. 25 For a fully discrete 10-year term life insurance policy on (x), you are given:
Copyright 2012 by Krzysztof Ostaszewski. All rights reserved. No reproduction in any form is permitted without explicit permission of the copyright owner.

(i) Death benefits are 100,000 plus the return of all gross premiums paid without interest. (ii) Expenses are 50% of the first years gross premium, 5% of renewal gross premiums and 200 per policy expenses each year. (iii) Expenses are payable at the beginning of the year. 1 = 0.17094. (iv) Ax :10 (v) ( IA )x:10 = 0.96728. x:10 = 6.8865. (vi) a
1

Calculate the gross premium using the equivalence principle. A. 3200 B. 3300 C. 3400 D. 3500 E. 3600

Solution. Let us write G for the gross premium sought. Using the equivalence principle, we write:
1 x:10 = 100,000 Ax x:10 + 200 a x:10 . Ga + G ( IA )x:10 + 0.45G + 0.05Ga :10 1 APV of death benefit APV of return of premium benefit % of premium expenses Per policy expenses

We solve for G and obtain 1 x:10 100,000 Ax + 200 a 100,000 0.17094 + 200 6.8865 :10 G= = 3604.2299. 1 0.95 6.8865 0.96728 0.45 x:10 ( IA )x:10 0.45 0.95 a Answer E.

Copyright 2012 by Krzysztof Ostaszewski. All rights reserved. No reproduction in any form is permitted without explicit permission of the copyright owner.

You might also like